Section 1.2: Row Reduction and Echelon Forms

  1. Solve the system whose augmented matrix is given as \( \left[\begin{array}{rrrr}{0} & {1} & {-4} & {4} \\ {1} & {-2} & {1} & {0} \\ {0} & {10} & {-10} & {10} \end{array}\right]\)     solution
  2. Solve the system whose augmented matrix is given by  \( \begin{bmatrix}     1 & -2 & 0 & 0 & 2\\ 0 & 0 & 1 & 0 & 3 \\ 0 & 0 & 0 & 1 & 4 \\ 0 & 0 & 0 & 0 & 0 \end{bmatrix} \)   solution
  3. Solve the system by converting the corresponding augmented matrix into a row or reduced row echelon form:   solution
  4. eqn
  5. Solve the system by converting the corresponding augmented matrix into a row or reduced row echelon form.  solution

  6. eqn2